Is this valid for improper integrals.

  • Thread starter Thread starter tnutty
  • Start date Start date
  • Tags Tags
    Integrals
Click For Summary
The discussion centers on the validity of using two limits in improper integrals, particularly regarding the integral of functions with singularities. It highlights that while some may combine limits into a single integral, this can lead to misleading results, as seen with the integral of 1/x, which diverges. The standard approach is to separate the integral at points of discontinuity, ensuring proper handling of singularities. The conversation also emphasizes that writing the integral as a principal value can obscure its true nature, especially when evaluating limits. Properly addressing singularities is crucial for accurate results in improper integrals.
tnutty
Messages
324
Reaction score
1
note: INT = integral and the above and bottom arguments are the bounds for the integral. infinity
INT [ f(x) ]
-infinity

=

lim a - >-infinity and lim b-> infinity of :
b
INT [f(x)] = pi
a

I am just wondering if the move having two limits in one integral is valid,
as done above. Or would I have to separate it
 
Last edited:
Physics news on Phys.org
I suppose it is, but usually we write it as:

\lim_{a \to \infty} \int_{-a}^a \arctan x dx
 
Oh, even better and more efficient, Thanks. To me its useless to split it into two integrals because there is not need.
 
Cyosis said:
I suppose it is, but usually we write it as:

\lim_{a \to \infty} \int_{-a}^a \arctan x dx
Not, that is NOT how we usually write it! That is the "principal value" of the integral and can be very misleading.

For example, if we write
\int_{-\infty}^\infty \frac{1}{x} dx
as
\lim_{a\rightarrow \infty} \int_{-a}^a \frac{1}{x}dx
it becomes
\lim_{a\rightarrow \infty} ln(|a|)- ln(|-a|)= 0

The standard way to write it is, in fact,
\lim_{a\rightarrow -\infty}\lim_{b\rightarrow \infty}\int_a^b f(x) dx
which, for f(x)= 1/x, does not exist.
 
I'm confused myself now, while I see it going wrong for the limit to infinity this raises a question for the interval (-1,1).

<br /> \lim_{a \to 1} \int_{-a}^a \frac{1}{x}dx=\lim_{a \to 1} \ln|a|-\ln|-a|=0<br />

Now do this again with double limits.
<br /> \lim_{a \to -1} \lim_{b \to 1} \int_a^b \frac{1}{x} dx=\lim_{a \to -1} \lim_{b \to 1} (\ln|b|-\ln|a|)=\ln|1|-\ln|-1|=0<br />

But this integral should diverge should it not? So given that the function has a singularity somewhere within its integration interval should we not also split the integral up in, in this case -1,0 and 0,1?

For example:

\lim_{a \to -1} \lim_{\epsilon \uparrow 0} \int_a^\epsilon \frac{1}{x}dx+\lim_{b \to 1} \lim_{\epsilon \downarrow 0} \int_\epsilon^b \frac{1}{x} dx
 
Last edited:
Question: A clock's minute hand has length 4 and its hour hand has length 3. What is the distance between the tips at the moment when it is increasing most rapidly?(Putnam Exam Question) Answer: Making assumption that both the hands moves at constant angular velocities, the answer is ## \sqrt{7} .## But don't you think this assumption is somewhat doubtful and wrong?

Similar threads

  • · Replies 19 ·
Replies
19
Views
2K
Replies
9
Views
1K
  • · Replies 9 ·
Replies
9
Views
2K
Replies
3
Views
2K
  • · Replies 105 ·
4
Replies
105
Views
6K
  • · Replies 5 ·
Replies
5
Views
2K
  • · Replies 5 ·
Replies
5
Views
2K
Replies
20
Views
2K
  • · Replies 22 ·
Replies
22
Views
3K
  • · Replies 15 ·
Replies
15
Views
2K